Tuyển tập Bất đẳng thức

pdf 235 trang Người đăng khoa-nguyen Lượt xem 1767Lượt tải 0 Download
Bạn đang xem 20 trang mẫu của tài liệu "Tuyển tập Bất đẳng thức", để tải tài liệu gốc về máy bạn click vào nút DOWNLOAD ở trên
Tuyển tập Bất đẳng thức
Mục lục
Lời nói đầu 4
Các thành viên tham gia biên soạn 5
1 Các bất đẳng thức kinh điển 6
1.1 Bất đẳng thức giữa trung bình cộng và trung bình nhân (AM-GM). . . . . . . . . 6
1.2 Bất đẳng thức giữa trung bình cộng và trung bình điều hoà (AM-HM). . . . . . . 6
1.3 Bất đẳng thức Cauchy - Schwarz. . . . . . . . . . . . . . . . . . . . . . . . . . . . 6
1.4 Bất đẳng thức Holder. . . . . . . . . . . . . . . . . . . . . . . . . . . . . . . . . . 7
1.5 Bất đẳng thức Chebyshev. . . . . . . . . . . . . . . . . . . . . . . . . . . . . . . . 7
1.6 Bất đẳng thức Minkowski. . . . . . . . . . . . . . . . . . . . . . . . . . . . . . . . 7
1.7 Bất đẳng thức Schur. . . . . . . . . . . . . . . . . . . . . . . . . . . . . . . . . . . 7
1.8 Bất đẳng thức Vornicu - Schur. . . . . . . . . . . . . . . . . . . . . . . . . . . . . 8
1.9 Bất đẳng thức Bernoulli. . . . . . . . . . . . . . . . . . . . . . . . . . . . . . . . . 8
1.10 Ba tiêu chuẩn SOS thường gặp. . . . . . . . . . . . . . . . . . . . . . . . . . . . . 9
2 Một số đánh giá quen thuộc 9
3 Tuyển tập bất đẳng thức 10
3.1 Bài 1.1 đến bài 1.40 . . . . . . . . . . . . . . . . . . . . . . . . . . . . . . . . . . . 10
3.2 Bài 2.1 đến bài 2.40 . . . . . . . . . . . . . . . . . . . . . . . . . . . . . . . . . . . 39
3.3 Bài 3.1 đến bài 3.40 . . . . . . . . . . . . . . . . . . . . . . . . . . . . . . . . . . . 59
3.4 Bài 4.1 đến bài 4.40 . . . . . . . . . . . . . . . . . . . . . . . . . . . . . . . . . . . 80
3.5 Bài 5.1 đến bài 5.40 . . . . . . . . . . . . . . . . . . . . . . . . . . . . . . . . . . . 104
3.6 Bài 6.1 đến bài 6.40 . . . . . . . . . . . . . . . . . . . . . . . . . . . . . . . . . . . 132
3.7 Bài 7.1 đến bài 7.40 . . . . . . . . . . . . . . . . . . . . . . . . . . . . . . . . . . . 148
3.8 Bài 8.1 đến bài 8.40 . . . . . . . . . . . . . . . . . . . . . . . . . . . . . . . . . . . 168
3.9 Bài 9.1 đến bài 9.40 . . . . . . . . . . . . . . . . . . . . . . . . . . . . . . . . . . . 193
3.10 Bài 10.1 đến bài 10.40 . . . . . . . . . . . . . . . . . . . . . . . . . . . . . . . . . 211
3
Lời nói đầu
Biển vẫn mãi nhấp nhô với những con sóng dạt vào bờ, thuyền vẫn mãi lênh đênh theo từng con
sóng đi vào đại dương, và trong đất liền cuộc sống vẫn có nhiều bất cập còn đang xảy ra,. . . , tất
cả những điều đó đều là các bất đẳng thức trong phạm trù đặc thù của từng lĩnh vực. Trong toán
học cũng vậy nói đến bất đẳng thức là chúng ta nói đến một lớp bài toán khó mà ẩn chứa bên
trong có nhiều lời giải đẹp lạ kì làm say đắm biết bao nhiêu người.
Trong thời đại công nghệ thông tin với việc kết nối internet bạn có thể giao lưu học hỏi được rất
nhiều về các phương pháp làm bài bất đẳng thức, hoặc học hỏi với nhiều cuốn sách về bất đẳng
thức đang bày bán trên thị trường nhưng để có một cuốn sách bất đẳng thức hay với sự hội tụ
tinh hoa kiến thức của nhiều người thì điều đó chính là điểm mạnh của cuốn sách bất đẳng thức
mà các bạn đang cầm trên tay.
"Tuyển Tập Bất Đẳng Thức" với khoảng bốn trăm bài toán bất đẳng thức chọn lọc được gửi tới
từ các bạn trẻ, các thầy cô giáo yêu toán trên mọi miền của tổ quốc, ở đó bao gồm các bài toán
bất đẳng thức mới sáng tạo, các bài toán bất đẳng thức khó, các bài toán bất đẳng thức hay và
thú vị mà các bạn trẻ muốn chia sẻ với mọi người. Điều đó tạo nên sự hấp dẫn, tính cập nhật và
thời đại của cuốn sách này.
Bạn đọc hãy nhâm nhi với những lời giải hay, những ý tưởng độc đáo, những sáng kiến lạ kì trong
cách giải từng bài toán để từ đó rút kinh nghiệm học tập cho mình, giúp cho bạn thêm yêu, thêm
tin vào việc giải nhiều bài toán bất đẳng thức.
Với tinh thần làm việc nghiêm túc, ham học hỏi nhóm biên tập xin được gửi lời cảm ơn sâu
sắc tới tất cả các bạn đã tham gia gửi bài và giải bài, đồng thời cũng xin bày tỏ sự cảm
ơn và kính trọng tới thầy giáo Châu Ngọc Hùng - THPT Ninh Hải - Ninh Thuận đã nhiệt
tình cố vẫn kĩ thuật latex. Nhóm biên tập cũng xin gửi lời cảm ơn tới ban quản trị diễn đàn
 đã cổ vũ, động viên anh em trong quá trình làm việc để
ngày hôm nay chúng ta có một cuốn sách hay, có giá trị cao về kiến thức chuyên môn mà lại hoàn
toàn miễn phí về tài chính.
"TUYỂN TẬP BẤT ĐẲNG THỨC" chính thức được phát hành trên cộng đồng mạng những
người yêu toán, để từ đó thổi một luồng gió mới đem lại nhiều điều mới lạ cho học sinh, là tài
liệu tham khảo hữu ích cho giáo viên trong việc giảng dạy và học tập bất đẳng thức.
Do thời gian gấp rút và trình độ có hạn, dù rất cố gắng song những sai sót là khó tránh khỏi rất
mong nhận được sự thông cảm, chia sẻ, góp ý của các bạn để nhóm biên tập hoàn thiện cuốn sách
tốt hơn. Mọi ý kiến đóng góp xin gửi về địa chỉ hoangquan9@gmail.
Thay mặt nhóm biên soạn, tôi xin chân thành cảm ơn!
Hà Nội, ngày 10 tháng 8 năm 2011
Đại diện nhóm biên soạn
Chủ biên
Hoàng Minh Quân-Batigoal
4
Các thành viên tham gia biên soạn
Nội dung
• Hoàng Minh Quân - THPT Ngọc Tảo - Hà Nội.
• Tăng Hải Tuân - THPT Nguyễn Đức Cảnh - TP. Thái Bình.
• Lê Đức Cảnh - THPT Chuyên Lê Hồng Phong-Nam Định.
• Đào Thái Hiệp - PTNK - ĐHQG HCM.
• Phạm Tuấn Huy - PTNK - ĐHQG HCM.
• Phạm Quang Hưng - THPT Cao Bá Quát - Hà Nội.
• Phạm Tiến Kha - THPT Chuyên Lê Hồng Phong - TP. HCM.
• Nguyễn Văn Khánh - THPT Chuyên Bắc Ninh - TP. Bắc Ninh.
• Nguyễn Thị Nguyên Khoa - THCS Nguyễn Tri Phương - TP. Huế.
• Mạc Đức Trí - Hải Dương.
LATEX
Hỗ trợ kĩ thuật Latex
1. Châu Ngọc Hùng - THPT Ninh Hải -Ninh Thuận.
2. Các thành viên trong nhóm biên soạn.
Trình bày bìa
Hoàng Minh Quân - THPT Ngọc Tảo - Hà Nội.
5
1 Các bất đẳng thức kinh điển
1.1 Bất đẳng thức giữa trung bình cộng và trung bình nhân (AM-
GM).
Nếu a1, a2, . . . , an là các số thực không âm, thì
a1 + a2 + . . .+ an ≥ n n√a1a2 . . . an.
Đẳng thức xảy ra khi và chỉ khi a1 = a2 = . . . = an.
1.2 Bất đẳng thức giữa trung bình cộng và trung bình điều hoà (AM-
HM).
Nếu a1, a2, . . . , an là các số thực dương, thì
a1 + a2 + . . .+ an
n
≥ n1
a1
+ 1
a2
+ . . .+ 1
an
.
Đẳng thức xảy ra khi và chỉ khi a1 = a2 = . . . = an.
Thực chất đây là một hệ quả trực tiếp của bất đẳng thức Cauchy - Schwarz. Hai trường hợp thường
được sử dụng nhất của bất đẳng thức này là khi n = 3 hay n = 4.
Với n = 3, ta có
a+ b+ c
3
≥ 31
a
+ 1
b
+ 1
c
,
1
a
+
1
b
+
1
c
≥ 9
a+ b+ c
.
Với n = 4, ta có
a+ b+ c+ d
4
≥ 41
a
+ 1
b
+ 1
c
+ 1
d
,
1
a
+
1
b
+
1
c
+
1
d
≥ 16
a+ b+ c+ d
.
1.3 Bất đẳng thức Cauchy - Schwarz.
Dạng sơ cấp của nó được phát biểu như sau:
Nếu a1, a2, . . . , an và b1, b2, . . . , bn là các số thực tuỳ ý, thì
(a1b1 + a2b2 + . . .+ anbn)
2 ≤ (a21 + a22 + . . .+ a2n)(b1 + b2 + . . .+ b2n).
Đẳng thức xảy ra khi và chỉ khi
a1
b1
=
a2
b2
= . . . =
an
bn
, trong đó ta sử dụng quy ước: nếu mẫu
bằng 0 thì tử cũng bằng 0.
Trong đánh giá trên, chọn ai =
xi√
yi
,bi =
√
yi với xi, yi ∈ R; yi > 0, ta thu được bất đẳng thức
Cauchy - Schwarz dạng phân thức:
Nếu x1, x2, . . . , xn là các số thực và y1, y2, . . . , yn, là các số thực dương, thì
x21
y1
+
x22
y2
+ . . .+
x2n
yn
≥ (x1 + x2 + . . .+ xn)
2
y1 + y2 + . . .+ yn
.
Đẳng thức xảy ra khi và chỉ khi
x1
y1
=
x2
y2
= . . . =
xn
yn
.
6
1.4 Bất đẳng thức Holder.
Cho xij (i = 1, 2, . . . ,m; j = 1, 2, . . . , n) là các số thực không âm. Khi đó ta có
m∏
i=1
(
n∑
j=1
xij
) 1
m
≥
n∑
j=1
(
m∏
i=1
x
1
m
ij
)
.
Tổng quát hơn, nếu p1, p2, . . . , pn là các số thực dương thoả mãn p1 + p2 + . . .+ pn = 1, thì
m∏
i=1
(
n∑
j=1
xij
)pi
≥
n∑
j=1
(
m∏
i=1
xpiij
)
.
1.5 Bất đẳng thức Chebyshev.
Cho hai dãy số thực a1 ≤ a2 ≤ . . . ≤ an và b1, b2, . . . , bn. Khi đó
1. Nếu b1 ≤ b2 ≤ . . . ≤ bn thì n
n∑
i=1
aibi ≥
(
n∑
i=1
ai
)(
n∑
i=1
bi
)
;
2. Nếu b1 ≥ b2 ≥ . . . ≥ bn thì n
n∑
i=1
aibi ≤
(
n∑
i=1
ai
)(
n∑
i=1
bi
)
.
1.6 Bất đẳng thức Minkowski.
Cho hai dãy số dương a1, a2, . . . , an và b1, b2, . . . , bn. Với mọi r ≥ 1, ta có[
n∑
i=1
(ai + bi)
r
] 1
r
≤
(
n∑
i=1
ari
) 1
r
+
(
n∑
i=1
bri
) 1
r
.
Trường hợp r = 2 là trường hợp thường được sử dụng nhất của bất đẳng thức Minkowski. Khi đó
ta có √√√√ n∑
i=1
(ai + bi)
2 ≤
√√√√ n∑
i=1
a2i +
√√√√ n∑
i=1
b2i .
1.7 Bất đẳng thức Schur.
Cho các số thực không âm a, b, c. Khi đó với mọi số thực dương r, ta có
ar(a− b)(a− c) + br(b− a)(b− c) + cr(c− a)(c− b) ≥ 0.
Đẳng thức xảy ra khi và chỉ khi a = b = c, hoặc a = 0 và b = c, hoặc các hoán vị tương ứng.
Hai trường hợp thường được sử dụng nhất của bất đẳng thức Schur là r = 1 và r = 2.
Với r = 1, ta có bất đẳng thức Schur bậc ba
a3 + b3 + c3 + 3abc ≥ ab(a+ b) + bc(b+ c) + ca(c+ a),
(a+ b+ c)3 + 9abc ≥ 4(a+ b+ c)(ab+ bc+ ca),
(b− c)2(b+ c− a) + (c− a)2(c+ a− b) + (a− b)2(a+ b− c) ≥ 0,
7
a2 + b2 + c2 +
9abc
a+ b+ c
≥ 2(ab+ bc+ ca),
a
b+ c
+
b
c+ a
+
c
a+ b
+
4abc
(a+ b)(b+ c)(c+ a)
≥ 2.
Với r = 2, ta thu được bất đẳng thức Schur bậc bốn
a4 + b4 + c4 + abc(a+ b+ c) ≥ ab(a2 + b2) + bc(b2 + c2) + ca(c2 + a2).
1.8 Bất đẳng thức Vornicu - Schur.
Với mọi số thực a, b, c và x, y, z ≥ 0, bất đẳng thức
x(a− b)(a− b) + y(b− c)(b− a) + z(c− a)(c− b) ≥ 0
đúng nếu một trong các điều kiện sau được thoả mãn
1. a ≥ b ≥ c và x ≥ y;
2. a ≥ b ≥ c và z ≥ y;
3. a ≥ b ≥ c và x+ z ≥ y;
4. a ≥ b ≥ c ≥ 0 và ax ≥ by;
5. a ≥ b ≥ c ≥ 0 và cz ≥ by;
6. a ≥ b ≥ c ≥ 0 và ax+ cz ≥ by;
7. x, y, z là độ dài ba cạnh của một tam giác;
8. x, y, z là bình phương độ dài ba cạnh của một tam giác;
9. ax, by, cz là độ dài ba cạnh của một tam giác;
10. ax, by, cz là bình phương độ dài ba cạnh của một tam giác;
11. Tồn tại một hàm lồi t : I → R+, trong đó I là tập xác định của a, b, c, sao cho x =
t(a), y = t(b), z = t(c).
1.9 Bất đẳng thức Bernoulli.
Nếu α ≥ 1 hoặc α ≤ 0 thì (1 + x)α ≥ 1 + αx, ∀x > −1.
Nếu 0 ≤ α ≤ 1 thì (1 + x)α ≤ 1 + αx, ∀x > −1.
8
1.10 Ba tiêu chuẩn SOS thường gặp.
Giả sử a ≥ b ≥ c và có: Sa(b − c)2 + Sb(c − a)2 + Sc(a − b)2 ≥ 0(Sa, Sb, Sc là các hàm chứa
biến a, b, c).
Khi đó bất đẳng thức đúng nếu thỏa mãn một trong các tiêu chuẩn.
1.Sb ≥ 0, Sb + Sc ≥ 0, Sb + Sa ≥ 0.
2.Với a, b, c > 0 thỏa mãn Sb ≥ 0, Sc ≥ 0, a2Sb + b2Sa ≥ 0.
3.Sb ≥ 0, Sc ≥ 0, Sa(b− c) + Sb(a− c) ≥ 0
2 Một số đánh giá quen thuộc
1 Với mọi số thực a, b, ta luôn có
2(a2 + b2) ≥ (a+ b)2
Chứng minh. Để ý rằng
2(a2 + b2)− (a+ b)2 = (a− b)2 ≥ 0,
do đó ta có điều phải chứng minh.
Đẳng thức xảy ra khi và chỉ khi a = b. 2
2 Với mọi số thực a, b, c, ta luôn có
a2 + b2 + c2 ≥ ab+ bc+ ca
Chứng minh. Để ý rằng
a2 + b2 + c2 − (ab+ bc+ ca) = 1
2
[(a− b)2 + (b− c)2 + (c− a)2] ≥ 0,
do vậy ta có điều phải chứng minh.
Đẳng thức xảy ra khi và chỉ khi a = b = c. 2
Lưu ý. Từ đánh giá này ta suy ra
(a+ b+ c)2 ≥ 3(ab+ bc+ ca),
và
3(a2 + b2 + c2) ≥ (a+ b+ c)2.
3 Với mọi số thực dương a, b, c, ta luôn có
1
a
+
1
b
+
1
c
≥ 9
a+ b+ c
Chứng minh. Đây là một kết quả đã được đề cập ở trên. Lời giải có thể sử dụng bất đẳng thức
AM-HM hoặc Cauchy - Schwarz. Đẳng thức xảy ra khi và chỉ khi a = b = c. 2
9
3 Tuyển tập bất đẳng thức
3.1 Bài 1.1 đến bài 1.40
1.1 Cho x, y, z là các số thực dương thỏa mãn x+ y + z = 1. Chứng minh rằng:
8x + 8y + 8z ≥ 4x+1 + 4y+1 + 4z+1
Lời giải. Đặt a = 2x, b = 2y, c = 2z. Khi đó điều kiện đã cho được viết lại thành
a, b, c > 0; abc = 2x+y+z = 64,
và ta cần chứng minh
a3 + b3 + c3 ≥ 4(a2 + b2 + c2).
Để ý rằng ta có đẳng thức
a3 + 32− 6a2 = (a− 4)2(a+ 2),
từ đó sử dụng giả thiết a > 0 ta suy ra a3 + 32 ≥ 6a2. Thiết lập các bất đẳng thức tương tự cho
b và c và cộng vế theo vế các bất đẳng thức thu được, ta có
a3 + b3 + c3 + 96 ≥ 6(a2 + b2 + c2).
Như vậy để kết thúc chứng minh ta cần chỉ ra rằng
6(a2 + b2 + c2) ≥ 4(a2 + b2 + c2) + 96,
hay 2(a2 + b2 + c2) ≥ 96. Tuy nhiên bất đẳng thức này đúng theo bất đẳng thức AM-GM cho ba
số:
2(a2 + b2 + c2) ≥ 2.3 3
√
a2b2c2 = 6
3
√
4096 = 96.
Như vậy phép chứng minh đến đây hoàn tất.2
1.2 Cho a, b, c là các số thực thoả mãn a ≥ 4, b ≥ 5, c ≥ 6 và a2 + b2 + c2 = 90. Tìm giá trị
nhỏ nhất của biểu thức:
P = a+ b+ c
Lời giải. Đặt a = m+ 4, b = n+ 5, c = p+ 6, khi đó m,n, p ≥ 0 và từ giả thiết a2 + b2 + c2 = 90
ta suy ra
m2 + n2 + p2 + 8m+ 10n+ 12p = 13.
Để ý rằng ta có đẳng thức sau
(m+ n+ p)2 + 12(m+ n+ p) = (m2 + n2 + p2 + 8m+ 10n+ 12p) + 2(mn+ np+ pm+ 2m+ n).
Đến đây ta sử dụng các giả thiết đã cho để có
(m+ n+ p)2 + 12(m+ n+ p) ≥ 13,
từ đó ta suy ra m+ n+ p ≥ 1. Thay m = a− 4, n = b− 5, p = c− 6 ta suy ra a+ b+ c ≥ 10 hay
P ≥ 16.
10
Cuồi cùng, với a = 4, b = 5, c = 7 (thoả mãn các điều kiện đã cho) ta có P = 16 nên ta kết luận
16 là giá trị nhỏ nhất của biểu thức P .
Phép chứng minh hoàn tất. 2
1.3 Cho x, y, z là các số thực thoả mãn xy + yz + 3zx = 1. Tìm giá trị nhỏ nhất của biểu
thức:
P = x2 + y2 + z2
Lời giải. Đặt a =
9 + 3
√
17
4
và b =
3 +
√
17
4
, khi đó a = 3b và a+ 1 = 2b2 = c =
13 + 3
√
17
4
. Áp
dụng bất đẳng thức AM-GM ta thu được các bất đẳng thức sau
x2 + b2y2 ≥ 2bxy,
by2 + z2 ≥ 2byz,
a(z2 + x2) ≥ 2azx.
Đến đây ta cộng vế theo vế các bất đẳng thức thu được để có
(a+ 1)(x2 + z2) + 2b2y2 ≥ 2b(xy + yz) + 2azx,
hay c(x2 + y2 + z2) ≥ 2b(xy + yz + 3zx). Từ đó ta thay các giá trị của xy + yz + 3zx, b và c để
được
P = x2 + y2 + z2 ≥
√
17− 3
2
.
Cuối cùng, với x = z =
1
4
√
17
và y =
√
13
√
17− 51
34
(thoả mãn giả thiết) thì P =
√
17− 3
2
nên ta
kết luận
√
17− 3
2
là giá trị nhỏ nhất của biểu thức P .
Phép chứng minh hoàn tất.2
1.4 Cho a, b, c là các số thực dương thoả mãn a+ b+ c = 1. Chứng minh rằng:
a7 + b7
a5 + b5
+
b7 + c7
b5 + c5
+
c7 + a7
c5 + a5
≥ 1
3
Lời giải. Trước hết ta có đẳng thức sau
2(a7 + b7)− (a2 + b2)(a5 + b5) = (a− b)2(a+ b)(a4 + a3b+ a2b2 + ab3 + b4),
do vậy từ giả thiết a, b ≥ 0 ta suy ra
a7 + b7
a5 + b5
≥ a
2 + b2
2
.
Hoàn toàn tương tự ta cũng có
b7 + c7
b5 + c5
≥ b
2 + c2
2
và
c7 + a7
c5 + a5
≥ c
2 + a2
2
. Đến đây ta cộng vế theo
vế ba bất đẳng thức thu được để có
a7 + b7
a5 + b5
+
b7 + c7
b5 + c5
+
c7 + a7
c5 + a5
≥ a2 + b2 + c2.
11
Như vậy để kết thúc chứng minh ta cần chỉ ra rằng
a2 + b2 + c2 ≥ 1
3
.
Tuy nhiên bất đẳng thức trên đúng do
a2 + b2 + c2 − 1
3
= a2 + b2 + c2 − (a+ b+ c)
2
3
=
(a− b)2 + (b− c)2 + (c− a)2
3
≥ 0.
Như vậy phép chứng minh đến đây hoàn tất.2
1.5 Cho a, b, c là các số thực dương. Chứng minh rằng:
b2c
a3(b+ c)
+
c2a
b3(c+ a)
+
a2b
c3(a+ b)
≥ 1
2
(a+ b+ c)
Lời giải. Ta áp dụng AM-GM cho ba số như sau:
b2c
a3(b+ c)
+
b+ c
4bc
+
1
2b
≥ 3 3
√
b2c
a3(b+ c)
.
(b+ c)
4bc
.
1
2b
=
3
2a
,
từ đó ta suy ra
b2c
a3(b+ c)
≥ 3
2a
− 3
4b
− 1
4c
.
Thiết lập hai bất đẳng thức tương tự và cộng lại, ta suy ra
b2c
a3(b+ c)
+
c2a
b3(c+ a)
+
a2b
c3(a+ b)
≥
(
3
2
− 3
4
− 1
4
)
(a+ b+ c) =
1
2
(a+ b+ c).
Phép chứng minh hoàn tất.2
1.6 Cho a, b, c là các số thực không âm. Chứng minh rằng:
(a+ b+ c)3 ≥ 6√3(a− b)(b− c)(c− a)
Lời giải. Bất đẳng thức ban đầu mang tính hoán vị giữa các biến nên không mất tính tổng quát,
ta giả sử a = max {a, b, c}.
Với a ≥ b ≥ c thì vế phải là biểu thức không dương, trong khi vế trái là biểu thức không âm nên
bất đẳng thức cần chứng minh hiển nhiên đúng. Do vậy ta xét trường hợp a ≥ c ≥ b. Khi đó bình
phương hai vế ta thu được bất đẳng thức tương đương sau:
(a+ b+ c)6 ≥ 108[(a− b)(b− c)(c− a)]2.
Để ý rằng các biến không âm, và với việc sắp thứ tự như trên thì
[(a− b)(b− c)(c− a)]2 = [(a− b)(c− b)(a− c)]2 ≤ (a− c)2a2c2.
Đến đây ta áp dụng bất đẳng thức AM-GM để có
4(a− c)2a2c2 = (a− c)2.2ac.2ac ≤ [(a− c)
2 + 2ac+ 2ac]3
27
=
(a+ c)6
27
,
từ đó ta suy ra
[(a− b)(b− c)(c− a)]2 ≤ (a+ c)
6
108
,
12
và như vậy ta đã chứng minh được bất đẳng thức ban đầu vì
(a+ b+ c)6 ≥ (a+ c)6 ≥ 108[(a− b)(b− c)(c− a)]2.
Phép chứng minh hoàn tất.2
1.7 Cho a, b, c là các số thực dương thoả mãn a+ b+ c =
1
a
+
1
b
+
1
c
. Chứng minh rằng:
2(a+ b+ c) ≥ √a2 + 3 +√b2 + 3 +√c2 + 3
Lời giải. Dễ thấy bất đẳng thức cần chứng minh tương đương với mỗi bất đẳng thức trong dãy
sau
(2a−
√
a2 + 3) + (2b−
√
b2 + 3) + (2c−
√
c2 + 3) ≥ 0,
a2 − 1
2a+
√
a2 + 3
+
b2 − 1
2b+
√
b2 + 3
+
c2 − 1
2c+
√
c2 + 3
≥ 0,
a2 − 1
a
2 +
√
1 +
3
a2
+
b2 − 1
b
2 +
√
1 +
3
b2
+
c2 − 1
c
2 +
√
1 +
3
c2
≥ 0.
Các bất đẳng thức trên đều mang tính đối xứng giữa các biến nên không mất tính tổng quát ta
hoàn toàn có thể giả sử a ≥ b ≥ c. Khi đó không khó để ta suy ra
a2 − 1
a
≥ b
2 − 1
b
≥ c
2 − 1
c
và
1
2 +
√
1 + 3
a2
≥ 1
2 +
√
1 + 3
b2
≥ 1
2 +
√
1 + 3
b2
.
Như vậy theo bất đẳng thức Chebyshev ta được
a2 − 1
a
2 +
√
1 + 3
a2
+
b2 − 1
b
2 +
√
1 +
3
b2
+
c2−1
c
2 +
√
1 +
3
c2
≥ 1
3
(∑ a2 − 1
a
)∑ 1
2 +
√
1 +
3
a2

Nhưng theo giả thiết ta lại có∑ a2 − 1
a
= (a+ b+ c)−
(
1
a
+
1
b
+
1
c
)
= 0
nên ta suy ra
a2 − 1
a
2 +
√
1 + 3
a2
+
b2 − 1
b
2 +
√
1 +
3
b2
+
c2 − 1
c
2 +
√
1 +
3
c2
≥ 0, và vì vậy bất đẳng thức đã cho
cũng đúng.
Phép chứng minh hoàn tất.2
1.8 Cho a, b, c là các số thực dương thoả mãn a+ b+ c = 3. Chứng minh rằng:
ab√
c2 + 3
+
bc√
a2 + 3
+
ca√
b2 + 3
≤ 3
2
13
Lời giải. Trước hết để ý rằng
ab+ bc+ ca− (a+ b+ c)
2
3
= −
[
(a− b)2 + (b− c)2 + (c− a)2
6
]
≤ 0,
do đó từ giả thiết ta suy ra ab+ bc+ ca ≤ 3. Như vậy
ab√
c2 + 3
≤ ab√
c2 + ab+ bc+ ca
=
ab√
(c+ a)(b+ c)
.
Đến đây ta áp dụng bất đẳng thức AM-GM để có
ab√
c2 + 3
≤ 1
2
(
ab
c+ a
+
ab
b+ c
)
.
Thiết lập hai bất đẳng thức tương tự và cộng lại, ta suy ra dãy các đánh giá sau
ab√
c2 + 3
+
bc√
a2 + 3
+
ca√
b2 + 3
≤ 1
2
[(
ab
c+ a
+
bc
c+ a
)
+
(
bc
a+ b
+
ca
a+ b
)
+
(
ca
b+ c
+
ab
b+ c
)]
,
ab√
c2 + 3
+
bc√
a2 + 3
+
ca√
b2 + 3
≤ a+ b+ c
2
,
từ đó với lưu ý a+ b+ c = 3 ta suy ra bất đẳng thức đã cho là đúng.
Phép chứng minh hoàn tất.2
1.9 Cho a, b, c là các số thực dương thay đổi bất kì. Chứng minh rằng:(
b+ c
a
+
c+ a
b
+
a+ b
c
)2
≥ 4(ab+ bc+ ca)
(
1
a2
+
1
b2
+
1
c2
)
Lời giải 1. Dễ thấy rằng bất đẳng thức ban đầu tương đương với mỗi bất đẳng thức trong dãy
sau
[ab(a+ b) + bc(b+ c) + ca(c+ a)]2 ≥ 4(a+ b+ c)(a2b2 + b2c2 + c2a2)∑
a2b2(a+ b)2 + 2abc[
∑
a(a+ b)(a+ c)] ≥ 4
{∑
a3b3 + abc[
∑
ab(a+ b)]
}
Tuy nhiên để ý rằng ∑
a2b2(a+ b)2 − 4(
∑
a3b3) =
∑
a2b2(a− b)2 ≥ 0
và
2abc[
∑
a(a+ b)(a+ c)]− 4
{
abc[
∑
ab(a+ b)]
}
= 2abc[a3 + b3 + c3 + 3abc−
∑
ab(a+ b)] ≥ 0,
do đó bất đẳng thức ban đầu là đúng. Phép chứng minh đến đây hoàn tất.2
Lời giải 2. Bất đẳng thức ban đầu mang tính hoán vị giữa các biến, nên không mất tính tổng
quát, ta giả sử b = max {a, b, c}.
Ta áp dụng bất đẳng thức AM-GM như sau(
b+ c
a
+
c+ a
b
+
a+ b
c
)2
=
[(
a
b
+
b
a
+
a
c
)
+
(
b
c
+
c
b
+
c
a
)]2
≥ 4
(
a
b
+
b
a
+
a
c
)(
b
c
+
c
b
+
c
a
)
.
14
Như vậy để kết thúc chứng minh, ta cần chỉ ra rằng(
a
b
+
b
a
+
a
c
)(
b
c
+
c
b
+
c
a
)
≥ (ab+ bc+ ca)
(
1
a2
+
1
b2
+
1
c2
)
.
Tuy nhiên bằng phép biến đổi tương đương ta được
(b− a)(b− c)
ca
≥ 0,
là một đánh giá đúng do ta đã giả sử b = max {a, b, c}.
Phép chứng minh đến đây hoàn tất.2
Lời giải 3. Bất đẳng thức ban đầu mang tính đối xứng giữa các biến nên không mất tính tổng
quát, ta giả sử b nằm giữa a và c.
Ta áp dụng bất đẳng thức AM-GM như sau:
4(ab+ bc+ ca)
(
1
a2
+
1
b2
+
1
c2
)
≤
[
ab+ bc+ ca
ca
+ ca
(
1
a2
+
1
b2
+
1
c2
)]2
.
Như vậy để kết thúc chứng minh, ta cần chỉ ra rằng
b+ c
a
+
c+ a
b
+
a+ b
c
≥ ab+ bc+ ca
ca
+ ca
(
1
a2
+
1
b2
+
1
c2
)
.
Thực hiện phép biến đổi tương đương ta được bất đẳng thức
(a− b)(b− c)
b2
≥ 0,
tuy nhiên đây lại là một đánh giá đúng do ta đã giả sử b nằm giữa a và c.
Phép chứng minh đến đây hoàn tất.2
Nhận xét. Lời giải đầu tiên không mang nhiều ý nghĩa lắm, vì nó đơn thuần chỉ là biến đổi tương
đương kèm theo một chút tinh ý trong sử dụng các đánh giá quen thuộc và cơ bản. Ở đây

Tài liệu đính kèm:

  • pdfTuyen_tap_bat_dang_thuc_boi_duong_HS_gioi.pdf